• Aucun résultat trouvé

Irrationality of quick convergent series

N/A
N/A
Protected

Academic year: 2021

Partager "Irrationality of quick convergent series"

Copied!
9
0
0

Texte intégral

(1)

J

AROSLAV

H

AN ˇ

CL

Irrationality of quick convergent series

Journal de Théorie des Nombres de Bordeaux, tome

8, n

o

2 (1996),

p. 275-282

<http://www.numdam.org/item?id=JTNB_1996__8_2_275_0>

© Université Bordeaux 1, 1996, tous droits réservés.

L’accès aux archives de la revue « Journal de Théorie des Nombres de Bordeaux » (http://jtnb.cedram.org/) implique l’accord avec les condi-tions générales d’utilisation (http://www.numdam.org/conditions). Toute uti-lisation commerciale ou impression systématique est constitutive d’une infraction pénale. Toute copie ou impression de ce fichier doit conte-nir la présente mention de copyright.

Article numérisé dans le cadre du programme Numérisation de documents anciens mathématiques

(2)

Irrationality

of

quick

convergent

series

par JAROSLAV

HAN010CL

RÉSUMÉ. On démontre une généralisation d’un résultat dû à Badea

concer-nant l’irrationnalité de certaines séries à convergence rapide.

ABSTRACT. We generalize a previous result due to Badea relating to the irrationality of some quick convergent infinite series.

There are many papers

concerning

the

irrationality

of infinite series.

Erdos

[4]

proved

that if the sequence of

positive integers

con-00

verges

quickly

to

infinity,

then the series

E

is an irrational number. ~=1

The author

[7]

defined the irrational sequences and

proved

criterion for them. Another result is due to Erdos and Strauss

[5] .

They

proved

that if

is a sequence of

positive integers

with o0

n 00

and

1,

then the

number £

1fan

is rational if and

only

n=1

if an+i =

an -- an

+ 1 holds for every n &#x3E; no . Sander

[8]

proved

that

if and are two sequences of

positive

integers

such that

= oo and &#x3E;

1,

then the 00

number

E

bnfan

is irrational. n=1

Finally

Badea

[1]

proved

that if and

{bn }°°_ 1

are two sequences

of

positive integers

such that &#x3E;

bn)an+I/an

+

1,

then the sum

00

E

is an irrational number. Later he

generalized

his result

( [2] ) .

n=i

In this paper we will

generalize

Badea’s result in another way and prove t he

fol lowing

theorem.

(3)

THEOREM. Let sequences

of positive

integers.

If there is a

natural number rri such that the

following

three

inequalities

hold

for

every n &#x3E; no

00

then the number A

= ~

irrational.

~=1

Proo f :

For the sake of

simplicity

we will suppose that

(1) - (3)

hold for every n.

(If

not, we define

a~

= = for every n =

(4)

1,2,...

and these two sequences and

satisfy

then our above

requirements.)

Let us denote

One can prove

by

induction that

and

hold for i =

0,1,... ,

m + 1.

(5)

for every natural number n. Then

(4) - (9~

imply

that &#x3E; and

An,i

&#x3E; 0

for every

positive integer n

and

z=0~1~"

m -1.

First we will prove that

and

secondly

(11)

implies

that there is a number c &#x3E; 0 such that

Using

the famous theorem of Stolz

(see

e.g.

(6~),

we obtain

On the other hand

(10),

(11), (12)

and Brun’s Theorem

(see

e.g.

(3~)

imply

the

irrationality

of the number A.

(6)
(7)

(13)

and

(2)

yield

(10).

Similarly

(13) (if

substitute + 1 instead of

m)

and

(3)

yield

(11).

Remark: If we

put

m = 0 in the main

theorem,

then we receive

bn

&#x3E;

1,

0 &#x3E; -an and

1) -

Thus

bn

&#x3E;

+1 and this is the famous theorem due to Badea

(see

e.g.

[1]).

Consequence

1: Let and be two sequences of

positive

integers.

If

00

hold for every n &#x3E; no, then the number A

= £

is irrational. n=1

Proof:

Let us put m = 1 in the main theorem. Then

(14)

is

(1), (15)

is

(2)

and

(16)

is

(3).

Consequence

2: Let be a sequence of

positive integers

such that

bl

&#x3E; 2 and

hold for every n &#x3E; no where k is a

positive integer.

Then the number

00

A

= L

is irrational.

n=1

Let us

put

an in consequence 1. Then

(17)

immediately implies

(15) and

This and

bl

&#x3E; 2

imply

that the sequence is

increasing.

Thus

(15)

(8)

Let us define the sequence of

nonnegative

integers

such that

(17)

implies

that

Substituting

(19)

for

(18)

we obtain the

equivalent inequality

(21)

with

Carrying

out the

equivalent

calculations

step

by

step,

we receive

Using

(20)

and the fact that &#x3E;

2)

is an

increasing

sequence, it

is

enough

to prove that

where K is a suitable constant.

(22)

is

equivalent

with

(17)

implies

that

Because of

(23),

it is

enough

to prove that

where

K,

is a suitable constant too. But

(24)

is true for every n &#x3E; no.

Thus

(18)

is

right

and the number A is irrational.

(9)

REFERENCES

[1]

C. Badea, The Irrationality of Certain Infinite Series, Glasgow. Math J. 29

(1987),

221-228.

[2]

C. Badea, A Theorem on Irrationality of Infinite Series and Applications, Acta Arith. LXII.4

(1993),

313-323.

[3]

V. Brun, A Theorem about Irrationality, Arch. for Math. og Naturvideskab Kris-tiana 31, ,

(1910),

3

(German).

[4]

P. Erdös, Some Problems and Results on the Irrationality of the Sum of Infinite Series, J. Math. Sci. 10

(1975), 1-7.

[5]

P. Erdös and E. G. Strauss, On the Irrationality of Certain Ahmes Series, J. Indian Math. Soc. 27

(1963),

129-133.

[6]

G. M. Fichtengolz, The Lecture of Differential and Integral Calculations, part I,

issue 6, Nauka, Moskva, 1966

(Russian).

[7]

J. Han010Dl, Criterion for Irrational Sequences, J. Num. Theory 43 n° 1 (1993), 88-92.

[8]

J. Sándor, Some Classes of Irrational Numbers, Studia Univ. Babes-Bolyai Math.

29 (1984), 3-12. Jaroslav HANCL Department of Mathematics University of Ostrava Dvorakova 7 701 03 Ostrava 1 Czech Republic

Références

Documents relatifs

Our imperfection raises interesting questions: how should I rationally react to the discovery, e.g., that I will never be able to liberate myself fully of a particular prejudice which

Mahler in [6] introduced the main method of proving the irrationality of sums of infinite series.. This method has been extended several times and Nishioka’s book [7] contains a

Finally, based on the method, the algebraic extension of the rational number field, we concluded that under special occasions, the root of the equation can’t be some rational

Among the classical results in t h e theory of absolutely convergent Fourier series are the following theorems [6, Vol.. Attempts to generalize these theorems have

Theorem 1 gives a sufficient condition, which under certain circumstances is necessary (Theorem 2)... ARKIV FOR

Section 2 contains an elementary lemma, really a reformulation of the well-known formula expressing the abscissa of convergence of (1) in terms of the

14 Secretary of the LKSM (komsomols = Communist youth movement) in Lithuania.. and then told them that “you will all survive, but they are going to kill me”. She was

Moreover, the semi-infinite linear programming method described in Section 7 shows that the best numerical results are obtained when the irreducible algebraic curves of low